0% found this document useful (0 votes)
60 views

UPSC Civil Services Main 1986 - Mathematics Calculus: Sunder Lal

This document contains the solutions to questions from the UPSC Civil Services Main 1986 exam on the topic of mathematics. The first question examines the differentiability of a piecewise defined function f(x) at x=0. It is shown that f(x) is differentiable at x=0 with f'(0)=0. The second part of the first question evaluates the limit of an integral expression involving f(x) and obtains the value -1/2. The second question uses Rolle's theorem to establish an inequality relationship between an-bn and nbn-1(a-b) where a>b and n>1. The third question evaluates the expression x2

Uploaded by

sayhigaurav07
Copyright
© © All Rights Reserved
Available Formats
Download as PDF, TXT or read online on Scribd
0% found this document useful (0 votes)
60 views

UPSC Civil Services Main 1986 - Mathematics Calculus: Sunder Lal

This document contains the solutions to questions from the UPSC Civil Services Main 1986 exam on the topic of mathematics. The first question examines the differentiability of a piecewise defined function f(x) at x=0. It is shown that f(x) is differentiable at x=0 with f'(0)=0. The second part of the first question evaluates the limit of an integral expression involving f(x) and obtains the value -1/2. The second question uses Rolle's theorem to establish an inequality relationship between an-bn and nbn-1(a-b) where a>b and n>1. The third question evaluates the expression x2

Uploaded by

sayhigaurav07
Copyright
© © All Rights Reserved
Available Formats
Download as PDF, TXT or read online on Scribd
You are on page 1/ 7

UPSC Civil Services Main 1986 - Mathematics

Calculus
Sunder Lal
Retired Professor of Mathematics
Panjab University
Chandigarh

January 16, 2010

Question 1(a) 1. A function f (x) is defined as follows:


( 1
e1 x2 sin x1 , x 6= 0
f (x) =
0, x=0

Examine whether or not f (x) is differentiable at x = 0.

2. If f 0 (x) exists and is continuous, find the value of

1 x
Z
lim (x 3y)f (y) dy
x0 x2 0

Solution.
f (x) f (0)
1. By definition, f 0 (0) = lim , if this limit exists.
x0 x
f (x) f (0) 1 1 1
lim = lim e1 x2 sin
x0 x x0 x x
1 12 1
= e lim 2 e x lim x sin
x0 x x0 x
1 1
= e lim tet 0 Letting t = 2
, also lim x sin = 0
t x x0 x
= 0 lim tet = 0
t

Thus f (x) is differentiable at x = 0, and f 0 (0) = 0.

1
2. Define
Z x
F (x) = (x 3y)f (y) dy
0
Z x Z x
= x f (y) dy 3 yf (y) dy
0 0
Z x Z x
0
F (x) = f (y) dy + xf (x) 3xf (x) = f (y) dy 2xf (x)
0 0
F 00 (x) = f (x) 2f (x) 2xf 0 (x) = f (x) 2xf 0 (x)
Note that F 00 (x) exists because f 0 (x) exists.
F (x)
The required limit is lim 2 . Since lim F (x) = 0, lim x2 = 0 and both are differen-
x0 x x0 x0
tiable, we can apply LHospitals rule to get
F (x) F 0 (x)
lim = lim
x0 x2 x0 2x

Again LHospitals rule applies, so


F (x) F 00 (x) f (x) 2xf 0 (x) f (0)
lim 2
= lim = lim =
x0 x x0 2 x0 2 2
since f 0 (x) exists and is continuous, so limx0 xf 0 (x) = 0.
Thus Z x
1 f (0)
lim 2 (x 3y)f (y) dy =
x0 x 0 2

Question 1(b) Use Rolles theorem to establish that under suitable conditions (to be stated)

f (a) f (b) f (a) f 0 ()
g(a) g(b) = (b a) g(a) g 0 () , a < < b

Hence or otherwise deduce the inequality


nbn1 (a b) < an bn < nan1 (a b)
where a > b and n > 1.
Solution. Let f (x), g(x) be continuous in the closed interval [a, b] and differentiable in the
open interval (a, b). Let

f (a) f (x) x a f (a) f (b)
F (x) =
g(a) g(x) b a g(a) g(b)
xa
= [f (a)g(x) g(a)f (x)] [f (a)g(b) g(a)f (b)]
ba
Thus F (x) is

2
1. continuous in the closed interval [a, b].
2. differentiable in the open interval (a, b)
3. F (a) = F (b) = 0.
Thus F (x) satisfies the requirements of Rolles theorem, consequently, there exists , a <
< b such that F 0 () = 0. But
1
F 0 () = [f (a)g 0 () g(a)f 0 ()] [f (a)g(b) g(a)f (b)]
ba
f (a) f 0 ()
1 f (a) f (b) = 0

= 0
g(a) g () b a g(a) g(b)

therefore
f (a) f (b) 0
= (b a) f (a) f 0 () , a < < b


g(a) g(b) g(a) g ()
Let f (x) = xn , g(x) = 1, then the above result implies
n n n
b a b n n1
1 1 = (a b) 1
, b < < a
0
or an bn = (a b)n n1 . Now since bn1 < n1 < an1 , we get
nbn1 (a b) < an bn < nan1 (a b), as required.
(ax3 + by 3 )n 2u 2u 2u
Question 1(c) If u = + xf ( xy ), find the value of x2 2 + 2xy + y2 2 .
3n(3n 1) x x y y
Solution. We use the result proved in 2006, question 2(b): If f (x, y) is a homogeneous
function of degree n possessing continuous partial derivatives of degree 2,
2f 2f 2
2 f
x2 + 2xy + y = n(n 1)f
x2 x y y 2
(ax3 + by 3 )n
Let v = , then v is homogeneous of degree 3n, so
3n(3n 1)
2
2 v 2v 2
2 v (ax3 + by 3 )n
x 2
+ 2xy +y 2
= 3n(3n 1) = (ax3 + by 3 )n
x x y y 3n(3n 1)
Let w = xf ( xy ), then w is homogeneous of degree 1, so
2w 2w 2
2 w
x2 + 2xy + y = 1(1 1)w = 0
x2 x y y 2
Now u = v + w, so adding the above equations we have
2u 2u 2
2 u
x2 + 2xy + y = (ax3 + by 3 )n
x2 x y y 2

3
Question 2(a) 1. Without evaluating the involved integrals, show that
1
Z x Z
t dt x dt
+ =0
1 1 + t2 1 t(1 + t2 )
Z a+T
2. If f (x) is periodic of period T , show that f (t) dt is independent of a.
a

Solution.
du t 1 u2 u
1. Let t = u1 , so that dt = 2
and 2
= 2
= , implying that
u 1+t u 1+u 1 + u2
Z x Z 1   Z 1
t dt x u du x du
2
= 2
2
= 2
1 1+t 1 1+u u 1 u(1 + u )

Thus 1
Z x Z
t dt x dt
+ =0
1 1 + t2 1 t(1 + t2 )
Z a+T
2. Define F (a) = f (t) dt. We shall prove that F (a) = F (b) for any b. In particular
Ra T
F (a) = F (0) = 0
f (t) dt which is independent of a.

Z b+T Z a+T
F (b) F (a) = f (t) dt f (t) dt
b a
Z b+T Z a+T Z b
= f (t) dt f (t) dt f (t) dt
b b a
Z b+T Z b Z b
= f (t) dt + f (t) dt f (t) dt
b a+T a
Z b+T Z b
= f (t) dt f (t) dt
a+T a
Z b Z b
= f (u + T ) du f (t) dt u+T =t
a a
Z b Z b
= f (u) du f (t) dt = 0 f (u + T ) = f (u)
a a

Thus F (b) = F (a) for all b.

4
Question 2(b) Find the volume of the solid generated by revolving one arc of x = a(t
sin t), y = a(1 cos t) about its base.
Solution. One arc is given by 0 t 2 and the base is the x-axis. Thus
Z 2
dx
V = y 2 dt
dt
Z0 2 Z 2 
2 2 3 t 3
= a (1 cos t) a(1 cos t) dt = a 2 sin2 dt
0 0 2
Z
3 t
= 8a 2 sin6 d ( = )
0 2
Z
2
= 32a3 sin6 d (sin( ) = sin , so double the integral and half the limit)
0
531
= 32a3 = 5 2 a3
642 2

Z 2 Z x  21
Question 2(c) Evaluate (xy)2 +2(x+y)+1 dx dy by using the transformation
0 0
x = u(1 + v), y = v(1 + u). Assume u, v are positive in the region concerned.
Solution.
1. The Jacobian of the transformation is

(x, y) 1 + v u
= =1+u+v >0
(u, v) v 1 + u
because u 0, v 0.
2. The region of integration in the xy-plane is the triangle bounded by the lines y = 0, y =
x, x = 2.
3. The region of integration in the uv-plane lies in the first quadrant as u 0, v 0.
Clearly
x=y u=v
x = 2 u(1 + v) = 2
y = 0 v = 0 ( 1 + u > 0)

(1, 1)
v-axis
u(1 + v) = 2

(1, 0) (2, 0) u-axis

5
The curve u(1 + v) = 2 meets the u-axis at (2, 0) and the line u = v at (1, 1). The
region of integration is bounded by v = 0, v = u, u(1 + v) = 2 and therefor consists
of the triangle with vertices (0, 0), (1, 0), (1, 1) and the portion bounded by u = 1, v =
0, u(1 + v) = 2, in which u varies from 1 to 2, and v varies from 0 to 2u u
.

4.
  12   21
(x y)2 + 2(x + y) + 1 = (u v)2 + 2(2uv + u + v) + 1
1
= (u2 + v 2 + 2uv + 2u + 2v + 1) 2 = (1 + u + v)1

Thus the product of the Jacobian and the integrand is 1.

Thus
Z 2 Z x  21
2
I = (x y) + 2(x + y) + 1 dx dy
0 0
2
Z 1 Z u Z 2 Z
u
1
= dv du + dv du
0 0 1 0
Z 2
1 2
= + du 1
2 1 u
1
= 2 log 2
2

Paper II

Question 3(a) Find the maximum and minimum values of f (x, y) = 7x2 + 8xy + y 2 where
x, y are constrained by the relation x2 + y 2 = 1.

Solution. Let F (x, y) = 7x2 +8xy +y 2 +(x2 +y 2 1), where is Lagranges undetermined
multiplier. For extreme values
F F
= 14x + 8y + 2x = 0, = 8x + 2y + 2y = 0
x y

Thus (7 + )x + 4y = 0, 4x + (1 + )y = 0. Since (x, y) 6= (0, 0) because x2 + y 2 = 1, it


follows that

7 + 4
= (7 + )(1 + ) 16 = 2 + 8 9 = 0 = 9, 1
4 1 +

F F
x +y = 14x2 + 8xy + 2x2 + 8xy + 2y 2 + 2y 2 = 2f + 2(x2 + y 2 ) = 0
x y

6
Since x2 + y 2 = 1, we get f = at stationary points f = 9, 1. Thus the maximum
value of f is 9, minimum value is 1.
Check: We have found the maximum and minimum values without finding the stationary
points.
1 2
= 9 x = 2y, x2 + y 2 = 1 5y 2 = 1 y = , x =
5 5
1 2
= 1 y = 2x, x2 + y 2 = 1 5x2 = 1 x = , y =
5 5

In case (1), f (x, y) = 28


5
+ 165
+ 15 = 9. In case (2) f (x, y) = 57 16
5
+ 54 = 1, confirming the
above.
Note: The question could also be done by substituting x = cos t, y = sin t, and then
f (x, y) = 7 cos2 t+8 cos t sin t+sin2 t, which is now a function of one variable. Differentiating
and letting the derivative be 0, we get 14 cos t sin t + 8(cos2 t sin2 t) + 2 sin t cos t = 0.
Let z = tan t, then 2z 2 + 3z 2 = 0 z = 2, 21 (x, y) = ( 15 , 25 ), ( 25 , 15 )
f (x, y) = 9, 1.

You might also like